When does the tensor product distribute over an infinite direct product?












1














It is well known that the tensor product of $R$-modules over some ring $R$ does not, in general, distribute over infinite direct products, an obvious example being $mathbb Z_p otimes_mathbb Z mathbb Q neq 0$. I also know that a sufficient condition for the tensor product to distribute is this.



What other sufficient conditions are there? For instance, it would seem intuitive that $mathbb Q otimes_mathbb Z prod_mathbb N mathbb Z cong prod_mathbb N mathbb Q$. But is it?










share|cite|improve this question




















  • 1




    Are you considering these as groups? Please edit the question to be more specific.
    – Shaun
    3 hours ago






  • 1




    @Shaun Thanks for your comment. The isomorphism should be one of $mathbb Z$-modules, in the example at the end. I'll edit accordingly.
    – foaly
    2 hours ago
















1














It is well known that the tensor product of $R$-modules over some ring $R$ does not, in general, distribute over infinite direct products, an obvious example being $mathbb Z_p otimes_mathbb Z mathbb Q neq 0$. I also know that a sufficient condition for the tensor product to distribute is this.



What other sufficient conditions are there? For instance, it would seem intuitive that $mathbb Q otimes_mathbb Z prod_mathbb N mathbb Z cong prod_mathbb N mathbb Q$. But is it?










share|cite|improve this question




















  • 1




    Are you considering these as groups? Please edit the question to be more specific.
    – Shaun
    3 hours ago






  • 1




    @Shaun Thanks for your comment. The isomorphism should be one of $mathbb Z$-modules, in the example at the end. I'll edit accordingly.
    – foaly
    2 hours ago














1












1








1







It is well known that the tensor product of $R$-modules over some ring $R$ does not, in general, distribute over infinite direct products, an obvious example being $mathbb Z_p otimes_mathbb Z mathbb Q neq 0$. I also know that a sufficient condition for the tensor product to distribute is this.



What other sufficient conditions are there? For instance, it would seem intuitive that $mathbb Q otimes_mathbb Z prod_mathbb N mathbb Z cong prod_mathbb N mathbb Q$. But is it?










share|cite|improve this question















It is well known that the tensor product of $R$-modules over some ring $R$ does not, in general, distribute over infinite direct products, an obvious example being $mathbb Z_p otimes_mathbb Z mathbb Q neq 0$. I also know that a sufficient condition for the tensor product to distribute is this.



What other sufficient conditions are there? For instance, it would seem intuitive that $mathbb Q otimes_mathbb Z prod_mathbb N mathbb Z cong prod_mathbb N mathbb Q$. But is it?







abstract-algebra number-theory modules tensor-products infinite-product






share|cite|improve this question















share|cite|improve this question













share|cite|improve this question




share|cite|improve this question








edited 2 hours ago









Shaun

8,507113580




8,507113580










asked 3 hours ago









foaly

467515




467515








  • 1




    Are you considering these as groups? Please edit the question to be more specific.
    – Shaun
    3 hours ago






  • 1




    @Shaun Thanks for your comment. The isomorphism should be one of $mathbb Z$-modules, in the example at the end. I'll edit accordingly.
    – foaly
    2 hours ago














  • 1




    Are you considering these as groups? Please edit the question to be more specific.
    – Shaun
    3 hours ago






  • 1




    @Shaun Thanks for your comment. The isomorphism should be one of $mathbb Z$-modules, in the example at the end. I'll edit accordingly.
    – foaly
    2 hours ago








1




1




Are you considering these as groups? Please edit the question to be more specific.
– Shaun
3 hours ago




Are you considering these as groups? Please edit the question to be more specific.
– Shaun
3 hours ago




1




1




@Shaun Thanks for your comment. The isomorphism should be one of $mathbb Z$-modules, in the example at the end. I'll edit accordingly.
– foaly
2 hours ago




@Shaun Thanks for your comment. The isomorphism should be one of $mathbb Z$-modules, in the example at the end. I'll edit accordingly.
– foaly
2 hours ago










2 Answers
2






active

oldest

votes


















3















For instance, it would seem intuitive that $mathbb Q otimes_mathbb Z prod_mathbb N mathbb Z cong prod_mathbb N mathbb Q$. But is it?




It is not. The natural map from the LHS to the RHS fails to be surjective; its image is the subgroup of $prod mathbb{Q}$ consisting of those sequences whose terms have a common denominator, and hence does not include, for example, $prod frac{1}{n}$.



In general, for modules over a commutative ring $k$, the tensor product $M otimes_k (-)$ preserves infinite products if $M$ is finitely presented projective. I had thought at some point about whether it suffices for $M$ just to be finitely presented, but I don't remember what the conclusion was off the top of my head.






share|cite|improve this answer

















  • 1




    To be pedantic, those groups actually are isomorphic (though not by the natural map), since both are $mathbb{Q}$-vector spaces of dimension $mathfrak{c}$. Of course, the right question is whether the natural map is an isomorphism.
    – Eric Wofsey
    1 hour ago





















1














Here is a general criterion. (All tensor products in this answer are over $R$.)




Theorem: Let $R$ be a ring and let $M$ be a right $R$-module. Then the following are equivalent.




  1. The functor $Motimes -$ preserves products: that is, for every family $(A_i)$ of left $R$-modules, the canonical map $Motimesprod A_ito prod Motimes A_i$ is an isomorphism.


  2. $M$ is finitely presented.





Proof: First, suppose $M$ has a finite presentation $$R^mto R^nto Mto 0$$ and let $(A_i)$ be any family of left $R$-modules. We then get a commutative diagram
$$require{AMScd}
begin{CD}
R^motimes prod A_i @>>> R^notimes prod A_i @>>> Motimes prod A_i @>>> 0\
@VV{}V @VV{}V @VV{}V \
prod R^motimes A_i @>>> prod R^notimes A_i @>>> prod Motimes A_i @>>> 0
end{CD}$$
whose rows are exact. Now note that $R^motimes prod A_icong (prod A_i)^m$ and $prod R^motimes A_icong prod A_i^m$, and our vertical map between them is easily seen to be the canonical isomorphism which interchanges the products. So the left vertical map is an isomorphism and similarly so is the middle vertical map. By the five lemma, it follows that the right vertical map is an isomorphism, and thus $Motimes -$ preserves products.



Conversely, suppose $Motimes-$ preserves products. In particular, then, the canonical map $$varphi: Motimes R^Mto M^M$$ is an isomorphism. Considering the identity map $id:Mto M$ as an element of the product $M^M$, we have $$id=varphi(sum m_i otimes f_i)$$ for some finite collection of elements $m_iin M$ and $f_i:Mto R$. Evaluating both sides of this equation at an element $min M$ we find $$m=sum m_if_i(m).$$ Thus every element of $M$ is a linear combination of the elements $m_i$, so $M$ is finitely generated.



Now let $$0to Kto Fto Mto 0$$ be a presentation of $M$ with $F$ a finitely generated free module. To conclude $M$ is finitely presented, we must show that $K$ is finitely generated. Now let $(A_i)$ be any family of flat left $R$-modules and consider the commutative diagram
$$require{AMScd}
begin{CD}
Kotimes prod A_i @>>> Fotimes prod A_i @>>> Motimes prod A_i @>>> 0\
@VV{}V @VV{}V @VV{}V \
prod Kotimes A_i @>{alpha}>> prod Fotimes A_i @>>> prod Motimes A_i @>>> 0
end{CD}$$

which again has exact rows. The right vertical map is an isomorphism by hypothesis and the middle vertical map is an isomorphism since $F$ is finitely generated and free. Moreover, the map $alpha$ is injective since the $A_i$ are flat. A simple diagram chase now shows that the left vertical map is surjective.



Thus the canonical map $Kotimes prod A_ito prod Kotimes A_i$ is surjective for any family of flat modules $(A_i)$. In particular, the canonical map $Kotimes R^Kto K^K$ is surjective, which we have seen above implies that $K$ is finitely generated.





By similar arguments, you can show that the canonical map $Motimes prod A_itoprod Motimes A_i$ is always a surjection iff $M$ is finitely generated. Indeed, the forward direction is already contained in the forward direction of the proof above, and the reverse direction is similar to the proof of the reverse direction above.






share|cite|improve this answer























    Your Answer





    StackExchange.ifUsing("editor", function () {
    return StackExchange.using("mathjaxEditing", function () {
    StackExchange.MarkdownEditor.creationCallbacks.add(function (editor, postfix) {
    StackExchange.mathjaxEditing.prepareWmdForMathJax(editor, postfix, [["$", "$"], ["\\(","\\)"]]);
    });
    });
    }, "mathjax-editing");

    StackExchange.ready(function() {
    var channelOptions = {
    tags: "".split(" "),
    id: "69"
    };
    initTagRenderer("".split(" "), "".split(" "), channelOptions);

    StackExchange.using("externalEditor", function() {
    // Have to fire editor after snippets, if snippets enabled
    if (StackExchange.settings.snippets.snippetsEnabled) {
    StackExchange.using("snippets", function() {
    createEditor();
    });
    }
    else {
    createEditor();
    }
    });

    function createEditor() {
    StackExchange.prepareEditor({
    heartbeatType: 'answer',
    autoActivateHeartbeat: false,
    convertImagesToLinks: true,
    noModals: true,
    showLowRepImageUploadWarning: true,
    reputationToPostImages: 10,
    bindNavPrevention: true,
    postfix: "",
    imageUploader: {
    brandingHtml: "Powered by u003ca class="icon-imgur-white" href="https://imgur.com/"u003eu003c/au003e",
    contentPolicyHtml: "User contributions licensed under u003ca href="https://creativecommons.org/licenses/by-sa/3.0/"u003ecc by-sa 3.0 with attribution requiredu003c/au003e u003ca href="https://stackoverflow.com/legal/content-policy"u003e(content policy)u003c/au003e",
    allowUrls: true
    },
    noCode: true, onDemand: true,
    discardSelector: ".discard-answer"
    ,immediatelyShowMarkdownHelp:true
    });


    }
    });














    draft saved

    draft discarded


















    StackExchange.ready(
    function () {
    StackExchange.openid.initPostLogin('.new-post-login', 'https%3a%2f%2fmath.stackexchange.com%2fquestions%2f3053569%2fwhen-does-the-tensor-product-distribute-over-an-infinite-direct-product%23new-answer', 'question_page');
    }
    );

    Post as a guest















    Required, but never shown

























    2 Answers
    2






    active

    oldest

    votes








    2 Answers
    2






    active

    oldest

    votes









    active

    oldest

    votes






    active

    oldest

    votes









    3















    For instance, it would seem intuitive that $mathbb Q otimes_mathbb Z prod_mathbb N mathbb Z cong prod_mathbb N mathbb Q$. But is it?




    It is not. The natural map from the LHS to the RHS fails to be surjective; its image is the subgroup of $prod mathbb{Q}$ consisting of those sequences whose terms have a common denominator, and hence does not include, for example, $prod frac{1}{n}$.



    In general, for modules over a commutative ring $k$, the tensor product $M otimes_k (-)$ preserves infinite products if $M$ is finitely presented projective. I had thought at some point about whether it suffices for $M$ just to be finitely presented, but I don't remember what the conclusion was off the top of my head.






    share|cite|improve this answer

















    • 1




      To be pedantic, those groups actually are isomorphic (though not by the natural map), since both are $mathbb{Q}$-vector spaces of dimension $mathfrak{c}$. Of course, the right question is whether the natural map is an isomorphism.
      – Eric Wofsey
      1 hour ago


















    3















    For instance, it would seem intuitive that $mathbb Q otimes_mathbb Z prod_mathbb N mathbb Z cong prod_mathbb N mathbb Q$. But is it?




    It is not. The natural map from the LHS to the RHS fails to be surjective; its image is the subgroup of $prod mathbb{Q}$ consisting of those sequences whose terms have a common denominator, and hence does not include, for example, $prod frac{1}{n}$.



    In general, for modules over a commutative ring $k$, the tensor product $M otimes_k (-)$ preserves infinite products if $M$ is finitely presented projective. I had thought at some point about whether it suffices for $M$ just to be finitely presented, but I don't remember what the conclusion was off the top of my head.






    share|cite|improve this answer

















    • 1




      To be pedantic, those groups actually are isomorphic (though not by the natural map), since both are $mathbb{Q}$-vector spaces of dimension $mathfrak{c}$. Of course, the right question is whether the natural map is an isomorphism.
      – Eric Wofsey
      1 hour ago
















    3












    3








    3







    For instance, it would seem intuitive that $mathbb Q otimes_mathbb Z prod_mathbb N mathbb Z cong prod_mathbb N mathbb Q$. But is it?




    It is not. The natural map from the LHS to the RHS fails to be surjective; its image is the subgroup of $prod mathbb{Q}$ consisting of those sequences whose terms have a common denominator, and hence does not include, for example, $prod frac{1}{n}$.



    In general, for modules over a commutative ring $k$, the tensor product $M otimes_k (-)$ preserves infinite products if $M$ is finitely presented projective. I had thought at some point about whether it suffices for $M$ just to be finitely presented, but I don't remember what the conclusion was off the top of my head.






    share|cite|improve this answer













    For instance, it would seem intuitive that $mathbb Q otimes_mathbb Z prod_mathbb N mathbb Z cong prod_mathbb N mathbb Q$. But is it?




    It is not. The natural map from the LHS to the RHS fails to be surjective; its image is the subgroup of $prod mathbb{Q}$ consisting of those sequences whose terms have a common denominator, and hence does not include, for example, $prod frac{1}{n}$.



    In general, for modules over a commutative ring $k$, the tensor product $M otimes_k (-)$ preserves infinite products if $M$ is finitely presented projective. I had thought at some point about whether it suffices for $M$ just to be finitely presented, but I don't remember what the conclusion was off the top of my head.







    share|cite|improve this answer












    share|cite|improve this answer



    share|cite|improve this answer










    answered 1 hour ago









    Qiaochu Yuan

    277k32581919




    277k32581919








    • 1




      To be pedantic, those groups actually are isomorphic (though not by the natural map), since both are $mathbb{Q}$-vector spaces of dimension $mathfrak{c}$. Of course, the right question is whether the natural map is an isomorphism.
      – Eric Wofsey
      1 hour ago
















    • 1




      To be pedantic, those groups actually are isomorphic (though not by the natural map), since both are $mathbb{Q}$-vector spaces of dimension $mathfrak{c}$. Of course, the right question is whether the natural map is an isomorphism.
      – Eric Wofsey
      1 hour ago










    1




    1




    To be pedantic, those groups actually are isomorphic (though not by the natural map), since both are $mathbb{Q}$-vector spaces of dimension $mathfrak{c}$. Of course, the right question is whether the natural map is an isomorphism.
    – Eric Wofsey
    1 hour ago






    To be pedantic, those groups actually are isomorphic (though not by the natural map), since both are $mathbb{Q}$-vector spaces of dimension $mathfrak{c}$. Of course, the right question is whether the natural map is an isomorphism.
    – Eric Wofsey
    1 hour ago













    1














    Here is a general criterion. (All tensor products in this answer are over $R$.)




    Theorem: Let $R$ be a ring and let $M$ be a right $R$-module. Then the following are equivalent.




    1. The functor $Motimes -$ preserves products: that is, for every family $(A_i)$ of left $R$-modules, the canonical map $Motimesprod A_ito prod Motimes A_i$ is an isomorphism.


    2. $M$ is finitely presented.





    Proof: First, suppose $M$ has a finite presentation $$R^mto R^nto Mto 0$$ and let $(A_i)$ be any family of left $R$-modules. We then get a commutative diagram
    $$require{AMScd}
    begin{CD}
    R^motimes prod A_i @>>> R^notimes prod A_i @>>> Motimes prod A_i @>>> 0\
    @VV{}V @VV{}V @VV{}V \
    prod R^motimes A_i @>>> prod R^notimes A_i @>>> prod Motimes A_i @>>> 0
    end{CD}$$
    whose rows are exact. Now note that $R^motimes prod A_icong (prod A_i)^m$ and $prod R^motimes A_icong prod A_i^m$, and our vertical map between them is easily seen to be the canonical isomorphism which interchanges the products. So the left vertical map is an isomorphism and similarly so is the middle vertical map. By the five lemma, it follows that the right vertical map is an isomorphism, and thus $Motimes -$ preserves products.



    Conversely, suppose $Motimes-$ preserves products. In particular, then, the canonical map $$varphi: Motimes R^Mto M^M$$ is an isomorphism. Considering the identity map $id:Mto M$ as an element of the product $M^M$, we have $$id=varphi(sum m_i otimes f_i)$$ for some finite collection of elements $m_iin M$ and $f_i:Mto R$. Evaluating both sides of this equation at an element $min M$ we find $$m=sum m_if_i(m).$$ Thus every element of $M$ is a linear combination of the elements $m_i$, so $M$ is finitely generated.



    Now let $$0to Kto Fto Mto 0$$ be a presentation of $M$ with $F$ a finitely generated free module. To conclude $M$ is finitely presented, we must show that $K$ is finitely generated. Now let $(A_i)$ be any family of flat left $R$-modules and consider the commutative diagram
    $$require{AMScd}
    begin{CD}
    Kotimes prod A_i @>>> Fotimes prod A_i @>>> Motimes prod A_i @>>> 0\
    @VV{}V @VV{}V @VV{}V \
    prod Kotimes A_i @>{alpha}>> prod Fotimes A_i @>>> prod Motimes A_i @>>> 0
    end{CD}$$

    which again has exact rows. The right vertical map is an isomorphism by hypothesis and the middle vertical map is an isomorphism since $F$ is finitely generated and free. Moreover, the map $alpha$ is injective since the $A_i$ are flat. A simple diagram chase now shows that the left vertical map is surjective.



    Thus the canonical map $Kotimes prod A_ito prod Kotimes A_i$ is surjective for any family of flat modules $(A_i)$. In particular, the canonical map $Kotimes R^Kto K^K$ is surjective, which we have seen above implies that $K$ is finitely generated.





    By similar arguments, you can show that the canonical map $Motimes prod A_itoprod Motimes A_i$ is always a surjection iff $M$ is finitely generated. Indeed, the forward direction is already contained in the forward direction of the proof above, and the reverse direction is similar to the proof of the reverse direction above.






    share|cite|improve this answer




























      1














      Here is a general criterion. (All tensor products in this answer are over $R$.)




      Theorem: Let $R$ be a ring and let $M$ be a right $R$-module. Then the following are equivalent.




      1. The functor $Motimes -$ preserves products: that is, for every family $(A_i)$ of left $R$-modules, the canonical map $Motimesprod A_ito prod Motimes A_i$ is an isomorphism.


      2. $M$ is finitely presented.





      Proof: First, suppose $M$ has a finite presentation $$R^mto R^nto Mto 0$$ and let $(A_i)$ be any family of left $R$-modules. We then get a commutative diagram
      $$require{AMScd}
      begin{CD}
      R^motimes prod A_i @>>> R^notimes prod A_i @>>> Motimes prod A_i @>>> 0\
      @VV{}V @VV{}V @VV{}V \
      prod R^motimes A_i @>>> prod R^notimes A_i @>>> prod Motimes A_i @>>> 0
      end{CD}$$
      whose rows are exact. Now note that $R^motimes prod A_icong (prod A_i)^m$ and $prod R^motimes A_icong prod A_i^m$, and our vertical map between them is easily seen to be the canonical isomorphism which interchanges the products. So the left vertical map is an isomorphism and similarly so is the middle vertical map. By the five lemma, it follows that the right vertical map is an isomorphism, and thus $Motimes -$ preserves products.



      Conversely, suppose $Motimes-$ preserves products. In particular, then, the canonical map $$varphi: Motimes R^Mto M^M$$ is an isomorphism. Considering the identity map $id:Mto M$ as an element of the product $M^M$, we have $$id=varphi(sum m_i otimes f_i)$$ for some finite collection of elements $m_iin M$ and $f_i:Mto R$. Evaluating both sides of this equation at an element $min M$ we find $$m=sum m_if_i(m).$$ Thus every element of $M$ is a linear combination of the elements $m_i$, so $M$ is finitely generated.



      Now let $$0to Kto Fto Mto 0$$ be a presentation of $M$ with $F$ a finitely generated free module. To conclude $M$ is finitely presented, we must show that $K$ is finitely generated. Now let $(A_i)$ be any family of flat left $R$-modules and consider the commutative diagram
      $$require{AMScd}
      begin{CD}
      Kotimes prod A_i @>>> Fotimes prod A_i @>>> Motimes prod A_i @>>> 0\
      @VV{}V @VV{}V @VV{}V \
      prod Kotimes A_i @>{alpha}>> prod Fotimes A_i @>>> prod Motimes A_i @>>> 0
      end{CD}$$

      which again has exact rows. The right vertical map is an isomorphism by hypothesis and the middle vertical map is an isomorphism since $F$ is finitely generated and free. Moreover, the map $alpha$ is injective since the $A_i$ are flat. A simple diagram chase now shows that the left vertical map is surjective.



      Thus the canonical map $Kotimes prod A_ito prod Kotimes A_i$ is surjective for any family of flat modules $(A_i)$. In particular, the canonical map $Kotimes R^Kto K^K$ is surjective, which we have seen above implies that $K$ is finitely generated.





      By similar arguments, you can show that the canonical map $Motimes prod A_itoprod Motimes A_i$ is always a surjection iff $M$ is finitely generated. Indeed, the forward direction is already contained in the forward direction of the proof above, and the reverse direction is similar to the proof of the reverse direction above.






      share|cite|improve this answer


























        1












        1








        1






        Here is a general criterion. (All tensor products in this answer are over $R$.)




        Theorem: Let $R$ be a ring and let $M$ be a right $R$-module. Then the following are equivalent.




        1. The functor $Motimes -$ preserves products: that is, for every family $(A_i)$ of left $R$-modules, the canonical map $Motimesprod A_ito prod Motimes A_i$ is an isomorphism.


        2. $M$ is finitely presented.





        Proof: First, suppose $M$ has a finite presentation $$R^mto R^nto Mto 0$$ and let $(A_i)$ be any family of left $R$-modules. We then get a commutative diagram
        $$require{AMScd}
        begin{CD}
        R^motimes prod A_i @>>> R^notimes prod A_i @>>> Motimes prod A_i @>>> 0\
        @VV{}V @VV{}V @VV{}V \
        prod R^motimes A_i @>>> prod R^notimes A_i @>>> prod Motimes A_i @>>> 0
        end{CD}$$
        whose rows are exact. Now note that $R^motimes prod A_icong (prod A_i)^m$ and $prod R^motimes A_icong prod A_i^m$, and our vertical map between them is easily seen to be the canonical isomorphism which interchanges the products. So the left vertical map is an isomorphism and similarly so is the middle vertical map. By the five lemma, it follows that the right vertical map is an isomorphism, and thus $Motimes -$ preserves products.



        Conversely, suppose $Motimes-$ preserves products. In particular, then, the canonical map $$varphi: Motimes R^Mto M^M$$ is an isomorphism. Considering the identity map $id:Mto M$ as an element of the product $M^M$, we have $$id=varphi(sum m_i otimes f_i)$$ for some finite collection of elements $m_iin M$ and $f_i:Mto R$. Evaluating both sides of this equation at an element $min M$ we find $$m=sum m_if_i(m).$$ Thus every element of $M$ is a linear combination of the elements $m_i$, so $M$ is finitely generated.



        Now let $$0to Kto Fto Mto 0$$ be a presentation of $M$ with $F$ a finitely generated free module. To conclude $M$ is finitely presented, we must show that $K$ is finitely generated. Now let $(A_i)$ be any family of flat left $R$-modules and consider the commutative diagram
        $$require{AMScd}
        begin{CD}
        Kotimes prod A_i @>>> Fotimes prod A_i @>>> Motimes prod A_i @>>> 0\
        @VV{}V @VV{}V @VV{}V \
        prod Kotimes A_i @>{alpha}>> prod Fotimes A_i @>>> prod Motimes A_i @>>> 0
        end{CD}$$

        which again has exact rows. The right vertical map is an isomorphism by hypothesis and the middle vertical map is an isomorphism since $F$ is finitely generated and free. Moreover, the map $alpha$ is injective since the $A_i$ are flat. A simple diagram chase now shows that the left vertical map is surjective.



        Thus the canonical map $Kotimes prod A_ito prod Kotimes A_i$ is surjective for any family of flat modules $(A_i)$. In particular, the canonical map $Kotimes R^Kto K^K$ is surjective, which we have seen above implies that $K$ is finitely generated.





        By similar arguments, you can show that the canonical map $Motimes prod A_itoprod Motimes A_i$ is always a surjection iff $M$ is finitely generated. Indeed, the forward direction is already contained in the forward direction of the proof above, and the reverse direction is similar to the proof of the reverse direction above.






        share|cite|improve this answer














        Here is a general criterion. (All tensor products in this answer are over $R$.)




        Theorem: Let $R$ be a ring and let $M$ be a right $R$-module. Then the following are equivalent.




        1. The functor $Motimes -$ preserves products: that is, for every family $(A_i)$ of left $R$-modules, the canonical map $Motimesprod A_ito prod Motimes A_i$ is an isomorphism.


        2. $M$ is finitely presented.





        Proof: First, suppose $M$ has a finite presentation $$R^mto R^nto Mto 0$$ and let $(A_i)$ be any family of left $R$-modules. We then get a commutative diagram
        $$require{AMScd}
        begin{CD}
        R^motimes prod A_i @>>> R^notimes prod A_i @>>> Motimes prod A_i @>>> 0\
        @VV{}V @VV{}V @VV{}V \
        prod R^motimes A_i @>>> prod R^notimes A_i @>>> prod Motimes A_i @>>> 0
        end{CD}$$
        whose rows are exact. Now note that $R^motimes prod A_icong (prod A_i)^m$ and $prod R^motimes A_icong prod A_i^m$, and our vertical map between them is easily seen to be the canonical isomorphism which interchanges the products. So the left vertical map is an isomorphism and similarly so is the middle vertical map. By the five lemma, it follows that the right vertical map is an isomorphism, and thus $Motimes -$ preserves products.



        Conversely, suppose $Motimes-$ preserves products. In particular, then, the canonical map $$varphi: Motimes R^Mto M^M$$ is an isomorphism. Considering the identity map $id:Mto M$ as an element of the product $M^M$, we have $$id=varphi(sum m_i otimes f_i)$$ for some finite collection of elements $m_iin M$ and $f_i:Mto R$. Evaluating both sides of this equation at an element $min M$ we find $$m=sum m_if_i(m).$$ Thus every element of $M$ is a linear combination of the elements $m_i$, so $M$ is finitely generated.



        Now let $$0to Kto Fto Mto 0$$ be a presentation of $M$ with $F$ a finitely generated free module. To conclude $M$ is finitely presented, we must show that $K$ is finitely generated. Now let $(A_i)$ be any family of flat left $R$-modules and consider the commutative diagram
        $$require{AMScd}
        begin{CD}
        Kotimes prod A_i @>>> Fotimes prod A_i @>>> Motimes prod A_i @>>> 0\
        @VV{}V @VV{}V @VV{}V \
        prod Kotimes A_i @>{alpha}>> prod Fotimes A_i @>>> prod Motimes A_i @>>> 0
        end{CD}$$

        which again has exact rows. The right vertical map is an isomorphism by hypothesis and the middle vertical map is an isomorphism since $F$ is finitely generated and free. Moreover, the map $alpha$ is injective since the $A_i$ are flat. A simple diagram chase now shows that the left vertical map is surjective.



        Thus the canonical map $Kotimes prod A_ito prod Kotimes A_i$ is surjective for any family of flat modules $(A_i)$. In particular, the canonical map $Kotimes R^Kto K^K$ is surjective, which we have seen above implies that $K$ is finitely generated.





        By similar arguments, you can show that the canonical map $Motimes prod A_itoprod Motimes A_i$ is always a surjection iff $M$ is finitely generated. Indeed, the forward direction is already contained in the forward direction of the proof above, and the reverse direction is similar to the proof of the reverse direction above.







        share|cite|improve this answer














        share|cite|improve this answer



        share|cite|improve this answer








        edited 20 mins ago

























        answered 55 mins ago









        Eric Wofsey

        179k12204331




        179k12204331






























            draft saved

            draft discarded




















































            Thanks for contributing an answer to Mathematics Stack Exchange!


            • Please be sure to answer the question. Provide details and share your research!

            But avoid



            • Asking for help, clarification, or responding to other answers.

            • Making statements based on opinion; back them up with references or personal experience.


            Use MathJax to format equations. MathJax reference.


            To learn more, see our tips on writing great answers.





            Some of your past answers have not been well-received, and you're in danger of being blocked from answering.


            Please pay close attention to the following guidance:


            • Please be sure to answer the question. Provide details and share your research!

            But avoid



            • Asking for help, clarification, or responding to other answers.

            • Making statements based on opinion; back them up with references or personal experience.


            To learn more, see our tips on writing great answers.




            draft saved


            draft discarded














            StackExchange.ready(
            function () {
            StackExchange.openid.initPostLogin('.new-post-login', 'https%3a%2f%2fmath.stackexchange.com%2fquestions%2f3053569%2fwhen-does-the-tensor-product-distribute-over-an-infinite-direct-product%23new-answer', 'question_page');
            }
            );

            Post as a guest















            Required, but never shown





















































            Required, but never shown














            Required, but never shown












            Required, but never shown







            Required, but never shown

































            Required, but never shown














            Required, but never shown












            Required, but never shown







            Required, but never shown







            Popular posts from this blog

            Quarter-circle Tiles

            build a pushdown automaton that recognizes the reverse language of a given pushdown automaton?

            Mont Emei